0 Daumen
423 Aufrufe



wie man weiß, liegt dem AKS-Primalitätstest die folgende Charakterisierung zugrunde:

(X + a)^n kongruent X^n + a mod n <=> n ist prim

Nun soll man die Umkehrung zeigen:

Wenn n nicht prim ist, gilt (X + a)^n nicht kongruent X^n + a mod n. Wie kann man diese Bahauptung zeigen?

Avatar von

Warum bezeichnest du 2. als Umkehrung?

1. hast du mit "genau dann wenn" -Pfeil versehen.

Sollte das stimmen, muss 2. auch "genau dann wenn" sein. Und du musst nichts beweisen.

Allerdings bezweifle ich 1.

1 Antwort

0 Daumen

Antwort mit KI erzeugt

AKS-Primalitätstestskriterium basiert auf \((X + a)^n \equiv X^n + a \mod n \Leftrightarrow n \) ist prim

Um zu zeigen, dass wenn \(n\) nicht prim ist, die Kongruenz \((X + a)^n \not\equiv X^n + a \mod n\) gilt, können wir allgemeine Eigenschaften von Primzahlen und den binomischen Lehrsatz verwenden.

Zunächst erinnern wir uns an den binomischen Lehrsatz, der besagt:

\( (X + a)^n = \sum_{k=0}^{n} \binom{n}{k} X^{n-k} a^k \)

Wenn \(n\) eine Primzahl ist, dann wissen wir durch den kleinen Satz von Fermat, dass für jedes \(a\) nicht durch \(n\) teilbar:

\( a^n \equiv a \mod n \)

Auch ist in dem Fall von \(n\) prim jede der Binomialkoeffizienten \(\binom{n}{k}\) für \(0 < k < n\) durch \(n\) teilbar, weil \(n\) im Zähler der Definition von \(\binom{n}{k}\) steht und \(n\) eine Primzahl ist (also keine Zahlen außer 1 und \(n\) selber \(n\) teilen). Das bedeutet, dass alle Terme \(\binom{n}{k} X^{n-k} a^k\) für \(0 < k < n\) ein Vielfaches von \(n\) sind und somit modulo \(n\) gleich 0 werden. Diese Tatsache lässt \((X + a)^n \) reduziert auf \(X^n + a\) modulo \(n\).

Doch wenn \(n\) nicht prim ist, scheitert diese Argumentation:

1. Binomialkoeffizienten: Nicht alle Binomialkoeffizienten \(\binom{n}{k}\) für \(0 < k < n\) müssen durch \(n\) teilbar sein, wenn \(n\) nicht prim ist. Dies bedeutet, dass einige Terme \(\binom{n}{k} X^{n-k} a^k\) nicht modulo \(n\) gleich 0 sind, und somit tragen diese Terme zu dem Ergebnis von \((X + a)^n \mod n\) bei.

2. Fall \(n\) zusammengesetzt: Für zusammengesetztes \(n\), können Fälle auftreten, wo \(a^n \not\equiv a \mod n\) für einige \(a\), was bedeutet, dass die Kongruenz \((X + a)^n \equiv X^n + a \mod n\) verletzt wird, da die Struktur von \((X + a)^n\) und \(X^n + a\) nicht gleich behandelt werden kann im Kontext der Modulararithmetik mit einem nicht-primen Modulus \(n\).

Um die Behauptung zu zeigen, betrachten wir ein spezifisches Beispiel, bei dem \(n\) nicht prim ist. Nehmen wir \(n = pq\) für zwei Primzahlen \(p\) und \(q\) und ein \(a\), das teilerfremd zu \(n\) ist. Ohne Verlust der Allgemeinheit könnte bei einigen \(a\) und bestimmten Werten von \(X\) die Kongruenz \((X + a)^n \equiv X^n + a \mod n\) verletzt sein, weil die Verteilung der Faktoren und Binomialkoeffizienten in \((X + a)^n\) nicht mehr sauber durch \(n\) kürzbar sind, um es auf die Form \(X^n + a\) zu bringen.

Dies verdeutlicht, warum die Originalaussage \((X + a)^n \equiv X^n + a \mod n \Leftrightarrow n\) ist prim eine notwendige und hinreichende Bedingung darstellt, da ihre Umkehrung das Versagen dieser sauberen Reduktion aufzeigt, wenn \(n\) nicht prim ist.
Avatar von

Ein anderes Problem?

Stell deine Frage

Willkommen bei der Mathelounge! Stell deine Frage einfach und kostenlos

x
Made by a lovely community